LSAT and Law School Admissions Forum

Get expert LSAT preparation and law school admissions advice from PowerScore Test Preparation.

 PB410
  • Posts: 39
  • Joined: Apr 01, 2017
|
#37706
Hi,

I'm not entirely sure why answer choice A is wrong.
I setup the stimulus to be
FPW-----> H and LC
and
(no)H or (no)LC----->FP(is not)W

is A wrong because it is dealing with a different sufficient condition than the one in the stimulus, because Ann kept her promise? I'm kind of struggling to understand the shift from Failure to keep a promise as a condition to someone who kept a promise as a condition.

D is set up as (no)H and (no)LC----->FP(no)W, which is an accurate contrapositive.
 Eric Ockert
PowerScore Staff
  • PowerScore Staff
  • Posts: 164
  • Joined: Sep 28, 2011
|
#37721
I think you hit the nail right on the head with the problem on answer (A). It's about keeping a promise rather than failing to keep a promise. Rethink this rule in the stimulus as just a rule ABOUT failed promises. Try this diagram:

Wrong :arrow: Hurt + Lose Confidence

And the contrapositive:

not Hurt OR not Lose Confidence :arrow: not Wrong

So answer (D) hits BOTH of the sufficient conditions in the contrapositive above (it only needed to hit one OR the other) and thus shows that the failure to keep a promise was not Wrong. Bingo.

Answer (A) falls outside the scope of the rule in the stimulus and so we really have no idea whether Ann's kept promise is wrong or not.

Hope that helps!
User avatar
 yenisey
  • Posts: 19
  • Joined: Oct 14, 2021
|
#96290
Why answer choice "C" is wrong?
User avatar
 katehos
PowerScore Staff
  • PowerScore Staff
  • Posts: 184
  • Joined: Mar 31, 2022
|
#96388
Hi yenisey!

If we diagram the principle contained within the stimulus, we get a diagram that looks like this:

Wrong :arrow: Harm & Lose Confidence
Harm or Lose Confidence :arrow: Wrong

This is the diagram we will use to find the correct answer choice! Looking at answer choice (C), we see a line of logic like this:
     Premise - George made a promise to Reiko
     Premise - George did not keep his promise to Reiko
     Premise - Reiko was hurt financially as a result
     Conclusion - Reiko financially hurt :arrow: George's failure to keep a promise was Wrong

Comparing this line of reasoning to our diagram, we can see that in it would be a Mistaken Reversal to conclude George is wrong because Reiko was hurt. Remember, the (plus this answer choice does not mention losing confidence, but, regardless, the MR is enough reason to eliminate (C) as a contender). Remember, presence of a necessary condition does not guarantee the existence of the sufficient!

I hope this helps :)
Kate

Get the most out of your LSAT Prep Plus subscription.

Analyze and track your performance with our Testing and Analytics Package.